Existenz von (V~,A~)(V~,A~)(\tilde{V},\mathbf{\tilde{A}}) für jedes (V,A)(V,A)(V,\mathbf {A}), das zu denselben EM-Feldern führt

Das elektrische Feld E und die magnetische Induktion B können in Bezug auf Potentiale parametrisiert werden v Und A :

E = v A T , B = × A .
Diese Parametrisierung ist nicht eindeutig, da wir eine Skalarfunktion finden können θ und definiere ein Paar ( v ~ , A ~ ) über v ~ = v θ / T Und A ~ = A + θ ( ) . Dann beides ( v , A ) Und ( v ~ , A ~ ) wird dasselbe hervorrufen ( E , B ) .

Über die Maxwell-Gleichungen können wir ein gekoppeltes System von Differentialgleichungen für finden v Und A :

{ A = μ J + ( A + ε μ v T ) v = ρ ε T ( A + ε μ v T ) , = 2 2 T 2 .

Diese können durch Betrachtung der Lorenz-Eichung, in die wir uns setzen, unabhängig gemacht werden A + ε μ v T = 0 . Wie kann man das für jeden explizit zeigen ( v , A ) Es gibt ( v ~ , A ~ ) (dh zu denselben Feldern führen), so dass dieses Paar die Lorenz-Eichbedingung erfüllt. Reicht es aus, die Ausdrücke zu betrachten ( ) und folgern Sie, dass beide Potentialpaare die Lorenz-Eichbedingung erfüllen müssen, was zu der Bedingung führt θ = 0 , dh wir können immer eine Skalarfunktion wählen θ wofür θ = 0 und über ein neues Potenzial nachdenken ( ) ?

Vielen Dank im Voraus.

Antworten (1)

Nehmen wir an, Sie haben ein Potentialpaar ( v , A ) die den Lorenz-Gauge nicht erfüllen. dh

A + 1 C 2 v T = F ( X , T ) 0.

Lassen Sie uns nun einen Messgerätwechsel auf etwas Neues durchführen ( v ' , A ' ) mit der Funktion θ ( X , T ) wie du erwähnt hast.

A ' = A + θ v ' = v θ T

Natürlich sind diese neuen Potentiale so konstruiert, dass sie auch die gleichen Felder erzeugen wie ( v , A ) , durch Eichinvarianz. Setzen wir diese Beziehungen in die obige Gleichung ein, erhalten wir

( A ' θ ) + 1 C 2 T ( v ' + θ T ) = F ( X , T ) ,

und Neuordnung, wir haben

A ' + 1 C 2 v ' T 2 θ + 1 C 2 2 θ T 2 = F ( X , T ) .

Daraus sollte klar sein, dass wenn die Funktion θ ( X , T ) die wir gewählt haben, erfüllt die Bedingung

2 θ + 1 C 2 2 θ T 2 = F ( X , T ) ,

Dann

A ' + 1 C 2 v ' T = 0.

Das Problem reduziert sich nun darauf, eine Funktion zu finden θ ( X , T ) die die obige Wellengleichung mit einer Quelle löst F ( X , j ) , und aus den Eigenschaften der Wellengleichung können wir immer eine solche finden θ ( X , T ) , sofern es "Quelle" ist F ( X , T ) ist nicht zu verrückt.


BEARBEITEN: Wie @hyportnex in den Kommentaren und in dieser Antwort darauf hinweist , muss man die Randbedingungen vollständig angeben, um eine Wellengleichung wie die obige zu lösen θ was das Lösen der Gleichung nicht trivial machen könnte, obwohl ich immer noch der Meinung bin, dass es immer eine Lösung geben sollte. Würde mich aber freuen, wenn mich jemand korrigieren könnte.

Im Vakuum ist das alles in Ordnung, aber wenn Materie vorhanden ist, müssen Sie auch bestimmte Kontinuitäts- / Diskontinuitäts-Randbedingungen erfüllen. Es ist nicht offensichtlich, dass diese für einige erfüllt werden können θ gegeben F mit 2 θ + 1 C 2 2 θ T 2 = F ( X , T ) .
@hyportnex Das ist interessant, das wusste ich nicht. Ich meine, klar θ müssten Randbedingungen vollständig spezifiziert werden, aber wie wirkt sich das Vorhandensein von Ladung oder Strom im Weltraum darauf aus? Wollen Sie damit sagen, dass es möglicherweise Ladungs- oder Stromkonfigurationen gibt, sodass das Lorenz-Messgerät nicht ausgewählt werden kann ?
Ich sage das nicht, ich sage nur, dass zu der von Ihnen zitierten Helmholtz-Gleichung einige Randbedingungen hinzugefügt werden müssen, und ich sehe das nicht als triviale Änderung an. siehe zum Beispiel physical.stackexchange.com/questions/535577/… , aber das muss relativistisch invariant gemacht werden.
Sehr interessant. Ich werde meine Antwort bearbeiten, um Ihren Punkt aufzunehmen.